Daniel Curry
01/19/2023 · High School

(a) The first five terms of a sequence are \( 8,3 / 5,2,-1 \) and -4 . (i) Find an expression, in terms of \( n \), for the \( n \)th term of the sequence. Answer ..............

UpStudy ThothAI Solution

Tutor-Verified Answer

Step-by-step Solution

To find an expression for the \( n \)th term of the sequence given the first five terms \( 8, \frac{3}{5}, 2, -1, -4 \), we first observe the pattern in the sequence. Let's denote the terms as follows: - \( a_1 = 8 \) - \( a_2 = \frac{3}{5} \) - \( a_3 = 2 \) - \( a_4 = -1 \) - \( a_5 = -4 \) Next, we can calculate the differences between consecutive terms to see if there is a pattern: 1. \( a_2 - a_1 = \frac{3}{5} - 8 = \frac{3 - 40}{5} = \frac{-37}{5} \) 2. \( a_3 - a_2 = 2 - \frac{3}{5} = \frac{10 - 3}{5} = \frac{7}{5} \) 3. \( a_4 - a_3 = -1 - 2 = -3 \) 4. \( a_5 - a_4 = -4 - (-1) = -3 \) Now, let's look at the second differences: 1. \( \frac{7}{5} - \frac{-37}{5} = \frac{7 + 37}{5} = \frac{44}{5} \) 2. \( -3 - \frac{7}{5} = -3 - \frac{7}{5} = \frac{-15 - 7}{5} = \frac{-22}{5} \) 3. \( -3 - (-3) = 0 \) The first differences do not seem to form a clear pattern, but the second differences suggest that the sequence might be quadratic. To find a quadratic expression, we can assume the \( n \)th term can be expressed as: \[ a_n = An^2 + Bn + C \] We can set up a system of equations using the first three terms: 1. For \( n = 1 \): \[ A(1^2) + B(1) + C = 8 \quad \Rightarrow \quad A + B + C = 8 \quad \text{(1)} \] 2. For \( n = 2 \): \[ A(2^2) + B(2) + C = \frac{3}{5} \quad \Rightarrow \quad 4A + 2B + C = \frac{3}{5} \quad \text{(2)} \] 3. For \( n = 3 \): \[ A(3^2) + B(3) + C = 2 \quad \Rightarrow \quad 9A + 3B + C = 2 \quad \text{(3)} \] Now we have the following system of equations: 1. \( A + B + C = 8 \) 2. \( 4A + 2B + C = \frac{3}{5} \) 3. \( 9A + 3B + C = 2 \) Next, we can eliminate \( C \) by subtracting equations: From (2) - (1): \[ (4A + 2B + C) - (A + B + C) = \frac{3}{5} - 8 \] \[ 3A + B = \frac{3}{5} - \frac{40}{5} = \frac{-37}{5} \quad \text{(4)} \] From (3) - (2): \[ (9A + 3B + C) - (4A + 2B + C) = 2 - \frac{3}{5} \] \[ 5A + B = 2 - \frac{3}{5} = \frac{10}{5} - \frac{3}{5} = \frac{7}{5} \quad \text{(5)} \] Now we have a new system of equations: 1. \( 3A + B = \frac{-37}{5} \) (4) 2. \( 5A + B = \frac{7}{5} \) (5) Subtract (4) from (5): \[ (5A + B) - (3A + B) = \frac{7}{5} - \frac{-37}{5} \] \[ 2A = \frac{7 + 37}{5} = \frac{44}{5} \] \[ A = \

Quick Answer

\( a_n = An^2 + Bn + C \) where \( A = -\frac{44}{10}, B = \frac{7}{5}, C = 8 \)
Answered by UpStudy AI and reviewed by a Professional Tutor
UpStudy ThothAI
Self-Developed and Ever-Improving
Thoth AI product is constantly being upgraded and optimized.
Covers All Major Subjects
Capable of handling homework in math, chemistry, biology, physics, and more.
Instant and Accurate
Provides immediate and precise solutions and guidance.
Try Now
Ask Tutors
Ask AI
10x
Fastest way to Get Answers & Solutions
By text

Enter your question here…

By image
Re-Upload
Uploaded Files
xxxx.png0%
Submit
📸 STUDY CAN BE A REAL STRUGGLE
Why Not UpStudy It?
Select your plan below
Premium

You can enjoy

  • Step-by-step explanations
  • 24/7 expert live tutors
  • Unlimited number of questions
  • No interruptions
  • Full access to answer and
    solution
Basic
  • Limited Solutions